Yahoo Answers is shutting down on May 4th, 2021 (Eastern Time) and beginning April 20th, 2021 (Eastern Time) the Yahoo Answers website will be in read-only mode. There will be no changes to other Yahoo properties or services, or your Yahoo account. You can find more information about the Yahoo Answers shutdown and how to download your data on this help page.

?
Lv 7
? asked in Science & MathematicsMathematics · 8 years ago

Can you apply Fermat's last theorem here?

Over on "Homework help" someone posted this question ...

Prove that there are no integers a,b and c that satisfy this equation.

a^3 + b^3 = 7c^3 +3

Now this seems to me to be an extension of Fermat's Last Theorem in the case n=3. Or am I missing something really obvious?

This ones got me stumped!

1 Answer

Relevance
  • 8 years ago
    Favorite Answer

    That's a really interesting thought. I tried to apply it for a few minutes and got stuck. It just not easy to go from the equation a^3 + b^3 = 7c^3 + 3 back to an equation with three integer cubes. Maybe it's doable, but I think you should stick with modular arithmetic, which gives a nice proof:

    The cubes (mod 7) are 1 and 6. This equation says that a certain sum of two cubes is congruent to 3 (mod 7). But this is impossible, since none of 1+1, 1+6, 6+1, 6+6 is congruent to 3 (mod 7).

    I hope this helps!

Still have questions? Get your answers by asking now.